Integral approximation and nim strategy


Assignment:

Q1. Two players take turns removing 1, 2, 3, or 4 objects from a set of 16 identical objects (without replacing them.) The winner is the player who removes the last object. Find a winning strategy for one of the players (the one who plays first or second).

Q2. If a and b are both greater than -1, find the  lim(n→∞)?n^(b-a)  (1a+2a+?+na)/(1b+2b+?+nb).

Provide complete and step by step solution for the question and show calculations and use formulas.

Solution Preview :

Prepared by a verified Expert
Mathematics: Integral approximation and nim strategy
Reference No:- TGS01921718

Now Priced at $20 (50% Discount)

Recommended (96%)

Rated (4.8/5)